Difference between revisions of "2019 AMC 12B Problems/Problem 22"

m (Problem: Added the set of solution)
(Redirected page to 2019 AMC 10B Problems/Problem 24)
(Tag: New redirect)
 
Line 1: Line 1:
==Problem==
+
#REDIRECT[[2019_AMC_10B_Problems/Problem_24]]
Define a sequence recursively by <math>x_0 = 5</math> and
 
 
 
<math>x_{n+1} = \frac{x_n^2 + 5x_n + 4}{x_n + 6}</math>
 
 
 
for all nonnegative integers <math>n</math>. Let <math>m</math> be the least positive integer such that <math>x_m \leq 4 + \frac{1}{2^{20}}</math>.
 
 
 
In which of the following intervals does <math>m</math> lie?
 
 
 
<math>\textbf{(A) }[9,26]\qquad\textbf{(B) }[27,80]\qquad\textbf{(C) }[81,242]\qquad\textbf{(D) }[243,728]\qquad\textbf{(E) }[729,\infty)</math>
 
 
 
==Solution==
 
 
 
==See Also==
 
{{AMC12 box|year=2019|ab=B|num-b=21|num-a=23}}
 

Latest revision as of 18:10, 14 February 2019